Quantcast
Channel: A question about covariant derivative on spinor field (Vierbein formalism) - Physics Stack Exchange
Browsing all 2 articles
Browse latest View live

Answer by Qmechanic for A question about covariant derivative on spinor field...

The spinor Laplacian is (1). The square (2) of the Dirac operator minus the spinor Laplacian (1) is a quarter of the scalar Levi–Civita curvature, cf. Weitzenböck-Lichnerowicz identity.

View Article



A question about covariant derivative on spinor field (Vierbein formalism)

Let's use Latin letters $a,b,c,\cdots$ for local Minkowski frame indices, and Greek letters $\mu,\nu\,\lambda,\cdots$ for coordinate indices.On one hand, we all know that the covariant derivatives for...

View Article
Browsing all 2 articles
Browse latest View live


Latest Images

<script src="https://jsc.adskeeper.com/r/s/rssing.com.1596347.js" async> </script>
<script src="https://jsc.adskeeper.com/r/s/rssing.com.1596344.js" async> </script>